Conservative Vector Fields

Introduction

Remember, we can generate a vector field by taking the gradient of a scalar potential function:

ϕ(x,y)F(x,y)=ϕ(x,y)potential functiongradient field\begin{array}{c c c} \phi (x,y) & \longrightarrow & \vec{F} (x,y) = \nabla \phi (x,y)\ \textrm{potential function} & & \textrm{gradient field} \end{array}

Now, start with a vector field; in some cases, we can work backwards and find that it is the gradient of some scalar function. This leads to the following definition: if a vector field can be written as the gradient of some potential function (i.e. if some function ϕ(x,y)ϕ(x,y) exists such that F(x,y)=ϕ(x,y)\vec{F}(x,y)=∇ϕ(x,y)) then this field is conservative.

Terminology

The term "conservative" is related to the concept of conservation of energy. Some forces are conservative (like gravity and magnetism), meaning that if you travel along a closed path through a gravitational or magnetic field, the initial and final energy will be equal (conserved). Some forces, however, are not conservative (the notable example being friction).

Testing to see if a field is conservative

In 22-D: a vector field F=f(x,y),g(x,y)\vec{F}=⟨f(x,y),g(x,y)⟩ is conservative if and only if fy=gxf_y=g_x.

In 33-D: a vector field F=f(x,y,z),g(x,y,z),h(x,y,z)\vec{F} = \langle f(x,y,z), g(x,y,z), h(x,y,z) \rangle is conservative if and only if fy=gxf_y=g_x, fz=hxf_z=h_x, and gz=hyg_z=h_y.

Determine whether or not F=y,x+y\vec{F} = \langle -y,x+y \rangle is conservative.

Solution

Since f(x,y)=yf(x,y)=−y and g(x,y)=x+yg(x,y)=x+y, fy=1f_y=−1 and gx=1g_x=1. Then, since fy = gxf_y\ \cancel=\ g_x, this field is not conservative.

Determine whether or not F=2xyz2,x2+2z,2y2xz\vec{F} = \langle 2xy-z^2,x^2+2z,2y-2xz \rangle is conservative.

Solution

f(x,y,z)=2xyz2fy=2x, fz=2zg(x,y,z)=x2+2zgx=2x, gz=2h(x,y,z)=2y2xzhx=2z, hy=2fy=gx,fz=hx, and gz=hy    Conservative\begin{aligned} f(x,y,z) = 2xy-z^2 &\longrightarrow f_y = 2x,\ f_z=-2z\ g(x,y,z) = x^2+2z &\longrightarrow g_x = 2x,\ g_z = 2\ h(x,y,z) = 2y-2xz &\longrightarrow h_x = -2z,\ h_y = 2\ \ &\longrightarrow f_y=g_x, f_z=h_x, \textrm{ and } g_z=h_y\ \ &\implies \ans{\textrm{Conservative}} \end{aligned}

  1. Test whether F=y,x\vec{F} = \langle -y,-x \rangle is conservative or not.

  2. Yes

  3. Test whether F=excosy,exsiny\vec{F} = \langle e^{-x}\cos y,e^{-x}\sin y \rangle is conservative or not.

  4. Yes

  5. Test whether F=yz,xz,xy\vec{F} = \langle yz,xz,xy \rangle is conservative or not.

  6. Yes

Finding the Potential Function

Suppose we start with a conservative vector field, and we want to know what its potential function is. We can work backward by integrating the component functions of the vector field.

For instance, suppose F=4x+y,x+2y\vec{F}=⟨4x+y,x+2y⟩. Then we want to find the potential function ϕ(x,y)ϕ(x,y) where F=ϕ(x,y)\vec{F}=∇ϕ(x,y), meaning that ϕx=4x+yϕ_x=4x+y and ϕy=x+2yϕ_y=x+2y. Start by integrating ϕxϕ_x with respect to xx: ϕ=4x+y dx=2x2+xy+c(y)\phi = \int 4x+y\ dx = 2x^2 + xy + c(y)

Notice that the constant of integration is a function of yy (constant with respect to xx).

Similarly, we can integrate ϕyϕ_y with respect to yy: ϕ=x+2y dy=xy+y2+c(x)\phi = \int x+2y\ dy = xy + y^2 + c(x)

Notice that the only term that didn't appear in the first integral is y2y^2, so that must be c(y)c(y): ϕ(x,y)=2x2+xy+y2.\phi(x,y) = 2x^2+xy+y^2.

Find the potential function ϕ(x,y,z)ϕ(x,y,z) if F=2xyz2,x2+2z,2y2xz\vec{F}=⟨2xy−z^2,x^2+2z,2y−2xz⟩.

Solution

Start by integrating ϕxϕ_x with respect to xx, noting that the constant of integration will be a function of yy and zz: ϕx=2xyz2ϕ=2xyz2 dx=x2yz2x+c(y,z)\begin{aligned} \phi_x &= 2xy-z^2\ \phi &= \int 2xy-z^2\ dx\ &= x^2y-z^2x+c(y,z) \end{aligned}

If we do the same with respect to yy and zz, we can find the missing terms: ϕy=x2+2zϕ=x2+2z dy=x2y+2yz+c(x,z)ϕz=2y2xzϕ=2y2xz dz=2yzxz2+c(x,y)\begin{aligned} \phi_y &= x^2+2z\ \phi &= \int x^2 + 2z\ dy\ &= x^2y + 2yz + c(x,z)\ \ \phi_z &= 2y-2xz\ \phi &= \int 2y-2xz\ dz\ &= 2yz - xz^2 + c(x,y) \end{aligned}

Combining all the terms that appear in the three integrals (include each term only once): ϕ=x2yxz2+2yz\ans{\phi = x^2y - xz^2 + 2yz}

  1. Find the potential function ϕ(x,y,z)\phi (x,y,z) if F=z,1,x\vec{F} = \langle z,1,x \rangle.

  2. ϕ(x,y,z)=xz+y\phi (x,y,z) = xz+y

  3. Find the potential function ϕ(x,y,z)\phi (x,y,z) if F=excosy,exsiny\vec{F} = \langle e^{-x}\cos y, e^{-x}\sin y \rangle.

  4. ϕ(x,y)=excosy\phi (x,y) = -e^{-x}\cos y

  5. Find the potential function ϕ(x,y,z)\phi (x,y,z) if F=yz,xz,xy\vec{F} = \langle yz,xz,xy \rangle.

  6. ϕ(x,y,z)=xyz\phi (x,y,z) = xyz

Fundamental Theorem for Line Integrals

For a line integral in a vector field, there is a fundamental theorem analogous to the well-known fundamental theorem of calculus:

A vector field F\vec{F} is conservative (i.e. F=ϕ(x,y)\vec{F}=∇ϕ(x,y)) if and only if CFT ds=CFr (t) dt=ϕ(B)ϕ(A),\int_C \vec{F} \cdot \vec{T} \ ds = \int_C \vec{F} \cdot \vec{r}\ '(t)\ dt = \phi (B) - \phi (A), where AA and BB are the starting and ending points of the path CC, respectively.

In physical terms, this means that the line integral is independent of path; this will come into play when we calculate work done by a conservative force (after the next example).

Suppose ϕ(x,y)=12(x2y2)\phi (x,y)=\dfrac{1}{2}(x^2-y^2), so F(x,y)=ϕ(x,y)=x,y\vec{F} (x,y) = \nabla \phi (x,y) = \langle x,-y \rangle. If A=(1,0)A=(1,0) and B=(0,1)B=(0,1), consider the following two paths from AA to BB:

C1=r1(t)=cost,sint  for  0tπ2  (quarter circle)C2=r2(t)=1t,t  for  0t1  (straight line)\begin{aligned} C_1 &= \vec{r}_1(t) = \langle \cos t, \sin t \rangle \ \textrm{ for } \ 0 \leq t \leq \dfrac{\pi}{2} \ \textrm{ (quarter circle)}\ C_2 &= \vec{r}_2(t) = \langle 1-t,t \rangle \ \textrm{ for } \ 0 \leq t \leq 1 \ \textrm{ (straight line)} \end{aligned}

Show that C1Fr (t) dt=C2Fr (t) dt\int_{C_1} \vec{F} \cdot \vec{r}\ '(t)\ dt = \int_{C_2} \vec{F} \cdot \vec{r}\ '(t)\ dt (the line integral is independent of path).

Solution

C1Fr1 (t) dt=0π/22sintcost dt=0π/2sin(2t) dt=12cos(2t)0π/2=1C2Fr2 (t) dt=01t1t dt=011 dt=1    C1Fr (t) dt=C2Fr (t) dt\begin{aligned} \int_{C_1} \vec{F} \cdot \vec{r}1\ '(t)\ dt &= \int_0^{\pi/2} -2\sin t \cos t \ dt\ &= \int_0^{\pi/2} -\sin (2t)\ dt\ &= \dfrac{1}{2} \cos (2t) \bigg|0^{\pi/2} = -1\ \ \int{C_2} \vec{F} \cdot \vec{r}2\ '(t)\ dt &= \int_0^1 t-1-t\ dt\ &= \int_0^1 -1\ dt = -1\ \ &\implies \ans{\int{C_1} \vec{F} \cdot \vec{r}\ '(t)\ dt = \int{C_2} \vec{F} \cdot \vec{r}\ '(t)\ dt} \end{aligned}

Application: Work

The work done on an object by moving through a conservative force field (like a magnetic field or a gravitational field) depends only on the starting and ending points, not on the path taken (based on the fundamental theorem). Basically, the positive and negative changes in energy cancel each other out along different paths (and if the object returns to its starting point, the net work will be zero, since the initial and final energies will be the same).

W=CFr (t) dt=ϕ(B)ϕ(A)W = \int_C \vec{F} \cdot \vec{r}\ '(t)\ dt = \phi(B) - \phi(A)

Suppose ϕ(x,y)=3x+x2yy3\phi (x,y)=3x+x^2y-y^3, so F(x,y)=ϕ(x,y)=3+2xy,x23y2\vec{F} (x,y) = \nabla \phi (x,y) = \langle 3+2xy,x^2-3y^2 \rangle. Find the work done on an object moving along the following path C:r(t)=etsint,etcostC: r⃗(t)=⟨e^t \sin t,e^t \cos t⟩ for 0tπ0≤t≤π.

Solution

First, is F\vec{F} conservative? If so, we can calculate the work done by subtracting the final potential from the initial potential.

fy=2x=gx     conservativef_y = 2x = g_x \implies \textrm{ conservative}

Therefore, W=CFr (t) dt=ϕ(B)ϕ(A)A :  when t=0,r=0,1B :  when t=π,r=0,eπW=ϕ(0,eπ)ϕ(0,1)=(eπ)3+1=e3π+1\begin{aligned} W &= \int_C \vec{F} \cdot \vec{r}\ '(t) \ dt = \phi(B) - \phi(A)\ &A \ : \ \textrm{ when } t=0, \vec{r} = \langle 0,1 \rangle\ &B \ : \ \textrm{ when } t=\pi, \vec{r} = \langle 0,-e^{\pi} \rangle\ \ W &= \phi \left(0,-e^{\pi}\right) - \phi (0,1) = -\left(-e^{\pi}\right)^3+1\ &= \ans{e^{3\pi}+1} \end{aligned}

  1. Find the work required to move an object along a line segment from (0,0)(0,0) to (2,4)(2,4) if F=x,2\vec{F}=⟨x,2⟩, measured in N.

  2. W=10 JW=10\ J

  3. Find the work required to move an object from (1,2,1)(1,2,1) to (2,4,6)(2,4,6) if F=x,y,z\vec{F}=⟨x,y,z⟩, measured in N.

  4. W=25 JW=25\ J